Jump to content

Search the Community

Showing results for tags 'time bubbles'.

  • Search By Tags

    Type tags separated by commas.
  • Search By Author

Content Type


Categories

  • Brandon and Book News
  • Events, Signings, & Giveaways
  • Columns and Features
  • Site News
  • Shardcast

Forums

  • 17th Shard
    • Introduce Yourself!
    • 17th Shard Discussion
    • The Coppermind Wiki
    • Arcanum Discussion
  • Brandon Sanderson
    • General Brandon Discussion
    • Events and Signings
    • Sanderson Fan Works
    • Arcanum, the Brandon Sanderson Archive
  • Spoiler Zone
    • The Sunlit Man (No Cosmere Spoilers)
    • The Sunlit Man (Cosmere Spoilers)
  • The Cosmere
    • Cosmere Q&A
    • Cosmere Discussion
    • Stormlight Archive
    • Mistborn
    • Other Cosmere
  • Non-Cosmere Works
    • Cytoverse
    • Other Non-Cosmere
    • The Wheel of Time
  • Related Works
    • Writing Excuses and Intentionally Blank
    • Reading Excuses
    • Sanderson Curiosities & Unpublished Works
    • TWG Archive
  • Community
    • General Discussion
    • Entertainment Discussion
    • Forum Games & Random Stuff
    • Creator's Corner
    • Role-Playing
    • Social Groups, Clans, and Guilds

Blogs

  • Chaos' Blog
  • Leinton's Blog
  • 17th Shard Blog
  • KChan's Blog
  • Puck's Blag
  • Brandon's Blog
  • The Name of your Blog
  • Darth Squirrely's Blog
  • Tales of a Firebug
  • borborygmus' Blog
  • Zeadman's Blog
  • zas678's Blog
  • The Basement
  • Addy's Avocations
  • Seshperankh's Blog
  • First time reading The Well Of Ascension
  • Zarepath's Blog
  • "I Have Opinions About Books"
  • Test
  • Which actors would you like to see playing the characters of Mistborn?
  • Drifted Mists
  • Jaron's Realm
  • Roshar Speculative Theories
  • ChrisHamatake's Blog
  • Paradox Flint's Blog
  • Deoradhan's Blog
  • Storm Blessed's Blog
  • Elwynn's Blog
  • firstRainbowRose's Blog
  • Rotabush ShardBlog
  • Hoid's Compendium
  • InterContinental Adventures
  • Claincy Creates
  • Theories, quotes, and details to keep it all straight.
  • WoR Thoughts and Questions
  • Blogfalcon
  • David Coppercloud's Blog
  • yurisses' notes and theories
  • Lark Adventures
  • LUNA's Poetry
  • Inspiration Board
  • Trying to be Useful for a Change
  • Cosmere Nerd Things
  • The Way of Toasters
  • An Elephant's Blog
  • Shhh Spoilers for Ronald.
  • Wyn's Adventures in Geekiness
  • Words With Ene
  • Dapper's Blog
  • Things to talk about, stuff to do
  • Zelly's Healthy-Accountability Blog
  • Dapper's Music Blog
  • GM Test Blog
  • Rhythm of War Liveblog
  • Zephy’s Art Blog
  • Axioms Idioms & Adages
  • Weather Reports
  • Unnecessarily Overcomplicated
  • 5
  • The Blog of Dubious Copyright Legality
  • Trutharchivist's Rambles
  • Xino's corner of insanity
  • The Perfect Space Opera
  • My Journey Through Roshar (A Liveblog)
  • Lost Metal Liveblog by ccstat
  • D&D campaign design.
  • My Depression Log
  • Story Ideas and Whatnot
  • deltarune AU concept.
  • How I Relate to Every Character in The Stormlight Archive
  • A thing
  • random jank and jabber.
  • FNF crem

Find results in...

Find results that contain...


Date Created

  • Start

    End


Last Updated

  • Start

    End


Filter by number of...

Joined

  • Start

    End


Group


Website URL


AIM


MSN


ICQ


Yahoo


Jabber


Skype


Member Title


Location


Interests

Found 13 results

  1. So, I had an interesting realization the other day. We know that the speed bubbles from bendalloy or cadmium allomancy attach themselves to reference frames, and speed things up or slow them down relative to that reference frame. We know that those reference frames can be ones that experience acceleration: case in point, whenever a bubble attaches itself to the reference frame of the ground, which is rotating about Scadrial's axis and orbiting the sun. And, from the perspective of general relativity, any frame in which there is a gravitational field is really no different from an accelerated frame. We also know, from descriptions of the experiences of characters within speed bubbles, that nothing weird happens in regards to their experiences of gravity. They feel the same weight inside a bubble as they would outside a bubble. When you drop something inside of a speed bubble, it falls to the ground at the rate you would expect to see outside the bubble: it doesn't float in the air or suddenly crash to the ground. (You might call this an extended version of Einstein's equivalence principle: you can't tell whether you are inside a speed bubble by doing measurements on the weights of objects.) All this means that, in order for there to not be any weird effects on the weight of objects inside speed bubbles, is that speed bubbles have to treat inertial forces and gravitational forces the same way, just as we would expect from general relativity. And that means that if Wayne managed to anchor a speed bubble inside a large spinning centrifuge, he could observe the centrifuge slowed down (relative to the rate of time flow within the bubble) while still experiencing the full centrifugal force. If he dropped something inside, it would "fall" towards the outside of the centrifuge, even though the apparent motion of the centrifuge is practically stopped. Does anyone know of any WoBs dealing with speed bubbles in accelerated frames, or in free-fall? Someone needs to ask Brandon what would happen if Wayne burned bendalloy while on a merry-go-round, or while skydiving.
  2. What do you think would happen if a Nicroburst used their power on a Cadmium misting? What about two at the same time? How much farther would this allow you to stretch time? If cadmium's burn rate depends on the time inside a bubble, this could easily be used to slow down time insanely, but how much?
  3. Preface: This question is quite nerdy and I don’t expect anyone to think this is interesting but be. So I’ve been thinking about how the Mistborn Modern Era would work, so as the 2nd era has shown mistborn is on track to follow earth like technology (e.g. guns, cars, etc.) so it’s quite clear that computers are going to become a part of life on scadrial at some point. My question is how would a timebubble (of any sort) effect the action of a computer. Specially how would a timebubble effect a fibre optic connection, from my understanding fibre optics work by flashing a light on and off in different pulses which corespond to 1s and 0s in binary if you have a timebubble around that connection could you scramble that data by compressing the intervals with timebubbles. If anyone smarter than me can shed some light on this that’s be great. Thank you for reading my super nerdy question
  4. Note: This is version 2.0, written in January 2017. It's primary purpose is to incorporate knowledge from the post-Bands of Mourning era, but I've also taken the liberty of re-writing/structuring some stuff, cleaning up a bit of phrasing, fixing the formatting/links that were hurt by the most recent site transition, etc. The original version of this post is in this archival post. Introduction: Time bubbles are fascinating. An essentially passive effect that has profound impacts on the very nature of reality. They're ripe for exploration and exploitation. But they're also complicated. Brandon isn't much of a physics/math guy, and he knows that; the passage of time is a pretty fundamental aspect to most physics and messing with it can get messy. Because of this time bubbles have a lot of asterisks and exceptions built into them. So our intuitions about what happens when you speed up or slow down a patch of space-time aren't going to align with what happens in the cosmere, and Words of Brandon are more necessary than usual. Though we have an unusually large reliance on authorial fiat for our fundamentals, we can use those building blocks to go far with our analysis/logic. Time bubbles follow their own consistent rules: There's no need to throw up our hands in despair just because we can't trust our initial intuitions. Now I'm an old bantha, but one advantage is that I kind of know... everything... about time bubbles. I'd be highly interested if anyone could come up with anything I didn't know we knew, at the very least. So in the interests of public knowledge, here is a thread that lays out everything I know (that I can remember I know) about time bubbles, along with some clearly-demarcated expansions into the theoretical where I think it safe. (xTc) Table of Contents: (you can word-search for sections by xY values) x0. A note on terms x1: Frame of reference x1.1: Anchoring x1.2: Hitting moving bubbles x1.3: Misc. x2: Entering and exiting time bubbles x2.1: How occupancy is determined x2.2: Jostling x2.3: Edge-case physical interaction x2.3.1: Occupancy of clothing and held objects x3: Bubble interaction x3.1: Subjective burn rates x3.2: Competing bubble effects x4: Reality of time bubble effects x5: Conscious control of bubble attributes x6: Effects of duralumin/nicrosil x7: Effects on various magics, aluminum x8: Faster than light (FTL) possibilities x8.1: How I think FTL would work with time bubbles x8.2: Mechallomancy and time bubbles x8.3: Disclaimer x9: Realmatics xCn: Conclusion xAr: WoB Archive (x0) A note on terms: I'm not too ashamed to admit that I've been responsible for creating/using some... unorthodox terminology over the years. Sometimes it's intentional as I develop a term to describe/clarify a new concept, other times I'm just mistaken. Nowhere is this more present than in my discussion of time bubbles: for example, no one in the books calls them time bubbles. They're all referred to as "speed bubbles", even cadmium bubbles. So here's a list of potentially problematic terms, including any canon ones that might be unclear if I come across them. This is a living list, so feel free to post asking me to add/clarify anything. Time bubble Also known in actual canon as a "speed bubble". The area of space-time affected by an Allomancer (or mechallomantic cube) burning either bendalloy or cadmium. Compression factor A term for the "speed" of a time bubble. Either how much it speeds up time (bendalloy) or how much it slows it down (cadmium) in relation to the normal flow of time. So a bendalloy bubble where every second on the outside corresponds to 10 on the inside would be have a compression factor of 10x, where the reversed cadmium bubble might be 1/10. Bubble anchors/anchoring Refers to the frame of reference that a time bubble is "still" relative to. Can refer to either an abstract frame of reference or a specific object the movement of which constitutes that frame of reference. Jostling Refers to how objects are jostled/refracted/deflected as they cross the borders of time bubbles. FTL Stands for "Faster than light", a term for viable interstellar travel mechanisms that bypass the need to spend years or decades going at sublight speeds through the vast emptiness of space Mechallomancy A term used to refer to the "mechanical allomancy" (/feruchemy) used by the Southern Scadrians, and first seen in Bands of Mourning (x1) Frame of reference: Frame of reference is one of the spots where things start getting weird with time bubbles. Time bubbles change both the flow of time and the way that objects move within their sphere of influence. The exact way they change these two factors, though, is dependent upon frame of reference. Look to this WoB, our earliest on the matter: Bubbles anchored by bond: It follows upon any degree of thought on this WoB that bubbles "not moving" actually means "at rest relative to the frame of reference of the surface of Scadrial where Wayne first cast it." That bubble is moving around the planet's axis at some absurd speed, around Scadrial's sun at some more absurd speed, around the galactic core at a different absurd speed, and outward from the origin of the universe at an even more absurd speed. And yet a Wax that is "moving faster" inside a speed bubble isn't all the sudden catapulted into space at a thousand meters a second. His movement through space is only accelerated relative to the the bubble's definition of "at rest". How the anchor that defines "at rest" is decided a key question, then. (x1.1) Anchoring: We've got a few WoB's on the question of how bubble frames of reference are determined, at least so far as how the bubble itself moves through space. Bubble anchor determined by what it cuts You'll notice that this stands in nearly direct contradiction to the other (much older, admittedly) WoB where a train was an example of where the bubble wouldn't stay anchored. Thus the tone of surprise on my part for this new WoB. In regards to how to reconcile the two, I'll quote myself: This new quote goes into quite a lot more detail, is unambiguous as to the bubble being on the train, and is newer, and so trumps the old one if they are in fact in conflict. I can't find it at the moment, but someone has suggested that a way to reconcile the two WoBs is that for the first Brandon was thinking of the big picture where the bubble is shortly ripped off the train by the "jarring". Reading the newer WoB, a charitable interpretation has "it's probably going to ruin your time bubble" occurring over the course of instants, rather than minutes or seconds. Myself I lean more towards this just being an evolution of how Brandon models bubble frame of reference, at least to some extent. Perhaps he got to thinking more deeply about why a bubble on the train intersecting the ground wouldn't work and decided to weaken a previous "well the ground intersecting the bubble would just instantly pull it out" to a weaker "the ground intersecting the bubble will just gradually pull it out", or the like. - Then we get into some other mechanisms for anchoring bubbles. Savants can anchor their bubbles to themselves Complexities of what affects anchoring We also got some nice in-book evidence from Bands of Mourning. So anchor-determination is complicated. We've got a combination of local cognitive polling, savantism, cognitive gymnastics, mass, and likely speed. I wouldn't be shocked if there weren't more factors at play. At a guess, I'd say that these factors are all in competition for each other. Perhaps a very fast but small object will beat out a somewhat-massive but slow one. Or a savant might have an easier time de-anchoring a bubble from a moving train than from the surface of Scadrial. The core takeaway here is that bubble anchors can be manipulated, through both gross physical interactions and more realmatically-based shenanigans. - It's implied in the scene where the gang is testing out the primer cube by tossing it around that the cube spits out a bendalloy bubble while its in mid-air, then proceeds to land on the ground near Marasi with the bubble still centered on it. This would imply that the cube either has some special ability to anchor bubble frames of reference or (as I'm about to suggest) that while the bubble was airborne it simply "defaulted" to being anchored to the cube, but when it landed it assumed a normal ground-based frame of reference. This supported by Marasi walking up and picking up the cube off the ground without any mention of the bubble shifting. This is by no means a sure thing, though, and also raises questions (which I will not attempt to address) about what would happen to Marasis if she's standing still on the ground and gets "hit" by a moving bubble. - As a general note, there is some possibility that the bubble's movement and its frame of reference could be de-coupled from one another. So a bubble could be moving North at 5 m/s while having a frame of reference that thinks it's heading South at twice that speed. This would do some fun things to the motion of captured objects, I think, but by "fun" I here mean "more complicated than I want to think about". I do not believe that this possibility is likely, from a purely literary standpoint, given the complications that would ensue from it and the fact that such de-coupling has never been suggested by either books or WoB. Also Occam's Razor. Assuming that the bubble gets its internal frame of reference from its own movement through space, we can simplify things by assuming that whatever anchor the bubble is "linked" to is what defines both frames of reference. So a bubble anchored to a spaceship (to choose an entirely random example...) would be exactly at rest from the frame of reference of the bubble; this would mean that the ship wouldn't travel faster through space, since the ship isn't moving so far as the bubble is concerned. (x1.2) Hitting moving bubbles So we have all this evidence that bubble-anchoring is a thing and that it's possible (if at least somewhat non-trivial) to manipulate a bubble's anchor using both realmatic and brute-force physical means. What does this mean for when moving bubbles intersect "still" (for certain definitions) objects? Moving bubble's effect on captured object movement: Aside: So this was a fun conversation to have with Peter. I broke out MSPaint more than once. Peter's answer essentially destroys some of my more high-level thoughts on what happens when something's hit by a moving time bubble. This thread of mine is wrong on more than one level. The first level is that I was wrong even within my model: by what I said then even bendalloy bubbles should shuttle objects backwards by some degree. The second level is my assuming relativity of reference frames, where from what Peter says it looks like there needs to be some other mechanism at work to decide how the cork is "really" moving. To summarize farther, if you toss a cork over a moving train that has a time bubble on it, then that cork's frame of reference essentially gets incorporated into the bubble's/train's. The cork is all "I'm moving straight across this here train" and the bubble is all like "okay, you do that". The cadmium bubble case is easiest to grasp here: upon intersection, it's as if the cork is caught in amber, slowly pushing its way straight north across a bubble of space at the same time as that bubble tows the cork farther and farther east. - By my reading, one interesting consequence of this lies in the path the cork takes over the top of the train. In a world without time bubbles, the train would be moving under the cork as it flew over it. So, looking from a vantage point above the train, we would see the cork cross over the edge of the train at point A, then over the opposite edge at some point B that's further towards the end (read: western part) of the train, as the train's been moving under the cork as it flew. So if it was high noon the shadow of the cork would have described a diagonal from south-east to north-west on the top of the train. But once time bubbles get introduced, things are a bit dicier. My own more relativistic model from before this WoB would show the cork following that same diagonal from A to B, just faster/slower than normal. So a speed bubble would see the cork popping at at point B before B drew level with the cork-thrower. This fell out as a natural consequence of the model. The new model we get from Peter doesn't allow such a thing. Read literally, the cork enters at A, then eventually exits directly north of that point at some point A' (with A' described by the intersection of the north part of the train with a line perpendicular to the edge of the train and intersecting A). The fact that the cork never ends up west of the train seems to necessitate that the cork exits at some point at or east of A'. This means that the lateral movement of the cork relative to the top of the train is eliminated by hitting the bubble. This has theoretically interesting consequences for projectiles (throwing a grenade to hit Miles in the face, accounting for the motion of the train, will fail if there's even a 1x compression factor bubble over him), but realistically deflection will probably play a larger role in such uncontrolled circumstances. The more relevant case here more controlled circumstances: could this lateral movement elimination property of time bubbles be used deliberately? An incredibly inefficient example that comes to mind (I believe I'm stealing this from someone else, but can't recall who) would be using time bubbles as a delivery mechanism for high-energy projectiles: Shoot a railgun into a very very slow cadmium bubble, then move the bubble's anchor to be right next to the enemy ship and once the railgun round wins free it's pointed exactly where you want it to be. -- I'm also a tad confused by Peter's "In fact, I think it's safe to assume that the train is always moving to the east faster than the thrower is throwing the cork to the north. In that case, both types of bubbles will always end up pushing the cork at least somewhat to the east" line. It seems logical that any non-infinite compression factor for a bendalloy bubble would result in the cork ending up at least a bit farther east than it would have without the bubble. I'm not sure what the relative speed of the cork/train really has to do with that. So either Peter misspoke or there's some deeper logic to relative speed of motion in the bubble vs. cork scenario so far as what frame of reference "wins". I'm inclined towards the former, myself. --- There is also the additional edge-case of what happens if that train-bubble intersected someone with their feet firmly planted on the ground, or if the cork had a string attached to it leading back to the cork-thrower. Both are beyond the scope I can address at the moment. - Here we have a later attempt of mine to get some more information on this front: Conservation of momentum on entrance tied to redshift-solution: I got nothin' here, just thought it was worth putting in. (x1.3) Misc.: There is also the issue of how time passes within the bubble. Regardless of whether an object's movement is changed, it's still going to experience the same weirdness with the passage of time. This introduces... weirdness as traditional models of movement as <unit of distance>/<unit of time> are a bit skewed by everything's experience of time flowing faster/slower with only passing relation to how their movement is affected. I'll discuss that weirdness further down. Beyond that, there are some small concerns for what the relativity of the passage of time has to say about all of this. I haven't addressed this directly in the past, but I don't think we need concern ourselves with it overmuch, as the scales of relative speed where this would matter are all such that objects are going to be almost immediately yanked out of the bubbles anyway. Not much time to experience a slightly altered flow of time. If I had to guess, though, I would imagine that time bubbles are frame of reference agnostic so far as the passage of time goes. So a speeding-by spaceship that experience 1 second for every 10 of ours would experience 10 for every 100 of ours in a 10x speed bubble anchored in our frame of reference, as opposed to 91 or something odd like that. (x2) Entering and exiting time bubbles: Ah, bubble-border problems. How I loathe them. This is still one of the less well-explored areas of time bubbles. Bubble boarders are... odd. This has been one of the more weirdish topics about bubbles since we first started talking about them. Gradually, though, we've been accumulating WoBs. Bubble borders static So borders are static and do not "distend" or stretch out objects or anything weird like that. You put up a bubble and that border is going to stay put as a nice little sphere, no moving or changing in shape or the like. This has some interesting impacts on some other aspects of time bubbles, particularly occupancy. As a general note, many of the oddities regarding how time bubbles behave have to do with transitions between being inside/outside of the bubble. Doylist rationale behind bubble mechanics: (emphasis added) This "loss of kinetic energy" is a general trend we can observe from the earliest days of time bubbles Loss of kinetic energy: -- Side note: Light. It's weird. But it's not weird for any good in-universe reason that we know of, and there likely isn't one. At core it's all a result of handwavium being burned because if you actually red/blue-shifted light properly really really weird and unhappy things would happen. Like microwaving people. Nothing more to see here, really. There is a bit more realmatic stuff coming out more recently, however. Bubbles changing speed of light considered, but not what happens: Light explanation a ways off So yes, there is a going to be a Realmatic workaround to make everything mechanically jive with the whole "let's not microwave people" thing, but we won't see it until we're all gray in the head. (x2.1) How occupancy is determined: So this brings us to talking about how we decide what's in or out of time bubbles. If you have half an object inside of it, is it going fast or slow or half and half? Excellent question. Glad I asked it . Bubble occupancy So objects are either in or out in their entirety. Moreover, this gives us that bubble occupancy is determined Cognitively, the whole "how an object views itself" shtick. If we're to follow the usual formula, the other "part" Brandon was talking about is just how other people view the object. This is interesting Realmatically, and also lets us get away with buildings not being torn down as their support beams shear off or stuff like that. Beyond that, it tells us that whether an object is included if its half-and-half really is "what sounds right", to some extent. It's a question of whether you/the object/observers would judge that an object is properly "in" some arbitrary space when parts of it are outside of that space's boundary. Beyond this Cognitive fun for determining "in-ness", living things are special: Living touch enough Included as soon as you touch: - These two sets of WoBs seem to run counter to each other. On the one hand, we're told that if a train is excluded from the influence of a bubble then the people inside of it are too. But the bubble isn't "distended" by the train—a fact we know because of the "static" WoB at the beginning of this section. So the people in the train are still technically in the bubble, still touching it. This should trigger the "if you're living and you touch a bubble you're included" clause. Given that the train is not holding the bubble out, it seems that just that being "in" an object which is not affected by a bubble is enough to keep an otherwise-wholly-encompassed person out of the effects of the bubble. Looking to the fact that "any living thing touching the bubble is affected by the bubble", then, it seems that in an important sense people within a train aren't "touching" the bubble. My conclusion from this, then, is that passengers in that case are not touching the bubble because they are counted as "part" of the train in the same way that the train's engines and individual cars are "part" of some larger train object. Like how there is both a bead for a door and a bead for a wall containing that door in Shadesmar. Actually, this needs its own thread to explore. Have a thread, with attendant conclusions: (x2.2) Jostling: We know that objects and people are "jostled" when they enter or exit time bubbles. It took us a bit (until Bands of Mourning, in fact) to nail down that both entrance and exit cause jostling for both types of bubbles, but through the first three Era 2 (AoL-era) books we got examples or in-text exposition that clarifies that "jostling" happens for both bendalloy and cadmium bubbles whether the object is entering or leaving it. Some book quotes on the phenomenon for reference, though references are scattered through the Era 2 books: There's also the possibility that jostling messes with sound, given this quote: In this case, there are several possibilities for why sound is disrupted, not least being that the incoming sounds just get their frequency messed with. Jostling is still a distinct possibility, though. - We know some things about how jostling works on a more general level. Jostling theoretically predictable, realistically chaotic: The deflection objects suffer when passing through time bubbles is also proportional to the bubble's compression factor: so a 1.5x time bubble would deflect objects less than a 15x one. Deflection proportional to compression factor of bubble Richochet effect Brandon's response to the punching question (which response I, of course, neglected to give my full attention to because of course) gives some hint as to what factors influence the jostling. His talk of "losing momentum" and use of the word "ricochet" both suggest that the effect is a function of the speed and/or mass of the object. One aspect of jostling we don't know is if the angle and/or force of this jostling is a function of the size or the speed of the exiting object. We also don't know if the angle that an object hits the edge of the bubble at matters. -- When all's said and done, we know frustratingly little about how jostling behaves upon that moment of exiting the bubble, or its deeper implications. We do have Brandon saying: Leaving bubbles causes unique/rare effect: He then laughed and said "That won't make any sense for 10 books" This leads me to believe that this might be related to the FTL travel. So something worth talking about is happening here. Something "unique or rare", in fact. (x2.3) Edge-case physical interaction: In terms of thought experiments, odd things happen when you have moving objects that are accelerated/decelerated by time bubbles when they run into objects that are too big to be included in them. What happens if you have someone on a train who's trying to "go" 10x faster because he's in a bendalloy bubble that the train doesn't acknowledge? Example 1: So Wayne is on a train, leaning against a wall in the direction that the train is traveling, going 60 mph. He casts a bendalloy bubble which takes the ground as its frame of reference. The bubble doesn't stay with him for long, but what happens while he's there is the concern. The bubble is tied to the planet, so it sees Wayne going 60 mph and tries to boost him to 600 mph relative to the rest of the world. Now the train is too big to be part of the bubble, so that wall Wayne is leaning against is still going normal speed, only 60 mph so far "rest" relative to the planet is considered. So bullet-Wayne is now trying to go 540 mph into a solid, unmoving wall for at least a few tenths of a second. Does he go kersplat? If he were in the bubble for longer (say it was bigger), would his corpse be actively pushing the train forward, what with it's non-slowing, continuous energy input? -- Now this example might not work because perhaps even a small bendalloy bubble within the train is still enough to trigger the train's "NO Wayne, don't go into the bubble!" response. So let's modify it: Example 2: So Wax is steel jumping along outside a train, going the same speed as it in an essentially straight line. Let's say he runs into a (stationary, relative to the planet) speed bubble. If he's above the train, then he'll zip along for a moment and fall out, having moved ahead relative to the train. Same if he's next to the train or far enough behind it. But what if Wax is, say, one foot behind the train at this point in time? If Wax were alongside the train, he'd find himself a bit ahead of its caboose when he exited the bubble. But in this case he's behind it, and now approaching the train at 60 mph. Is he going to faceplant into the train? It seems he should, really. And there's no justification to say he's "on" the train, since he's flying along outside of it has been for quite awhile. Yet it seems odd, as in the earlier example, for someone affected by a bubble to directly collide with an object not affected by it, where this occurs collision solely because Wax is being affected by the bubble and the train isn't. -- This problem exists in many forms, especially if you start going all frame of reference on it. I do not have rock-steady and fully satisfying solution, but I think my "belonging" thread generated by the Occupancy section up above (x2.1) might give some insight into a possible answer. The concept is that Wax doesn't get splattered as soon as he hits the back of the train because as soon as he comes into contact with it he counts as being "on" it, or at the very least as a part of it. If the train is big/weighty/rooted enough that it isn't affected by the time bubble, then it's also weighty enough to pull Wax onto it immediately and cancel out the effects of the bubble. (x2.3.1) Occupancy of clothing and held objects: This is likely the exact same way that clothing and/or held items behave for time bubbles. While I have my whole thread on the matter, I have yet to address what happens to clothing when people touch time bubbles. I think it intuitive and natural that their clothing is included in the speedup/slowdown as well. It wouldn't do to have a "pulled out of his socks" situation it a man poked a time bubble with his finger while running by. So perhaps the mans clothing is included in the direct, "this is part of the human's Cognitive aspect" sense. It's a bit more of an open question whether held-items (like weapons) would have the same privilege. However this "part of me"-ness is acquired, there's a good intuitive case to be made that it's near-instantaneous, at least so far as bubble occupancy is concerned. If Wax sticks an arm out to grab a gun a moment before the rest of him flies through the bendalloy bubble, we wouldn't necessarily expect that gun to rip itself out of his hand. We wouldn't expect individual nuts and bolts on a train to try and fly off as the they ran through a cadmium bubble just because they were put onto the train during maintenance yesterday. We wouldn't even necessarily expect spare parts lying on the floor of the train, still waiting to be installed, to have such a "fly off into the air" reaction. Another point to make is that I would think a man wearing gloves who touched the edge of a bubble with only the gloves might get included, as the gloves are a part of him. That one's a tad in the air, though, as to whether the "part of me" extends so deeply. We wouldn't expect someone whose shirt got cut by a Shardblade to feel pained by it, after all. (x3) Bubble interaction: Time bubbles can overlap each other. This results in... odd things. First of all, they overlap "like a Venn diagram": Overlap Venn diagram Cancelling only in area of overlap: Moreover, their timey-wimey effects are multiplicative: Overlap multiplicative There was also an interesting question relatively recently about whether nested bubbles directly interfered with each other's borders. It was RAFO'd, sadly. Nested bubble size interaction: Then we have the world's largest RAFO culled from reddit: RAFO on nested bubbles: So that's all fine and dandy. They overlap multiplicatively like Venn diagrams. This results in a little oddness with subjective burn rates (as expanded on just below), but it all makes a decent amount of sense on its own. The problem arises when bubbles don't share the same (approximately, at least) frame of reference. (x3.1) Subjective burn rates: In the normal course of events, Marasi is going to burn x grams per second of cadmium relative to her timeframe. So even if you put shuffle her in and out of various time bubbles, she'll always have the experience of burning cadmium at the same rate. Subjective burn rates There's still the question of whether bubblers somehow manage to get extra energy when they're enveloped by slow bubbles. Myself I would guess "yes" (call it feeding off the energy of the slow bubble or something), if only because Peter was so insistent on nothing "weird" happening in the other case. (x3.2) Competing bubble effects: Note: This subsection in particular is nearly all rampant speculation, so take it with a grain of salt. Overlap with regards to time is a no-brainer. Maybe relativity makes things a tad odd here and there, but as a whole you just multiply things together (~X/1 for bendalloy bubbles and ~1/Y for Cadmium) until you get the rate that time flows. Movement, though, is a scary story. Very scary. I have a big fat thread on the matter. It turns out that it's wrong in several of the details due to the PAFO-clarification WoP about the cork up above. The general idea is still sound, I think: I think it likely that how an object's movement through space is altered by time bubbles is a function of taking the movement vectors and compression factors of overlapping bubbles into account and multiplying them together to get a single vector of movement for the encompassed object. i.e., two speed bubbles with the same compression factor moving at right angles to each other will see an object moving off at 45 degrees as they try to drag the object along with them. Objects assume this altered vector of movement for as long as they're encompassed, and then resume their normal inertia upon exiting the bubbles. If they exit one bubble and are still within another, then simply stop accounting for that first bubble's effects on our object's movement. Keep doing this until you're not in any bubbles at all and congratulations, you're back in real time/space. This raises some questions in regards to how "real" the movement-altering effects of time bubbles are. Given all the talk we have from Peter and Brandon about time bubbles actually adding/removing kinetic energy from objects (like this one), we can fairly safely conclude that time bubbles actually quite actively invest and rob objects of kinetic energy going in various directions. This gives some credence, I think, to the idea that all we have to do is some vector math to figure out how overlapping bubbles parse it out. (x4) Reality of time bubble effects: It seems there is a subset of the community who have some degree of doubt over the "reality" of time bubbles, at least to some extent. Do time bubbles actually accelerate the passage of time, or do they just simulate it in part? If they just simulate it, to what degree do they do so? Myself I've always historically operated under the working assumption that the effects of time bubbles were fully "real" unless proven otherwise, but it's worth making sure of. Our first evidence of time bubble reality is Peter's age-old kinetic energy WoP, reproduced here for convenience: Loss of kinetic energy: This establishes the altered motion of objects within bubbles as "real" motion, not just some inertia-less scuttling about of objects. More recently, we've gotten a few more pieces of information. We know that the effects of aging and the passage of time can be accelerated by bendalloy bubbles: Accelerated aging (paraphrase): Watches need to be reset: The inverse also applies for cadmium bubbles. Cadmium hermit can time-capsule self: The most direct answer we have on the matter is sadly a paraphrase: Cadmium affects time, not perceptions (paraphrase): Together I believe these add up to a pretty firm weight on the "time bubbles really affect movement and time" side of the scale, but it's still and open question to an extent. (x5) Conscious control of bubble attributes: Exactly how much control "bubblers" have over their bubbles is a bit of an open question. We have a (somewhat vague, sorry about that, he was understandably tired/distracted and I was also tired) WoB addressing the matter directly: Degree of control, but only before casting My reading here is that Brandon's "not very much" at the beginning was directed at the "after the bubble is up" portion of the question, leaving us the still-not-very-promising "some discretion" at the end to apply to during the casting. Fear not, though! At the very least, this quote shows us that there is some degree of control over both compression factors and size at the moment of casting; added to this is the new fact that that "not very much" is a distinctly larger amount than "none" with regards to altering bubbles when they're up, though it's still in the air if it's entirely a matter of degree or if there's also differences in kind. Myself, I read the quote as saying that it's both, mainly with bubblers only being able to alter compression factors a bit via burn rate once the thing is up. - Later attempts to get more information bore little fruit. Bubble size/strength more controllable than shown, size not inverse to strength: - As a general overview, we know from the Era 2 books that bubbles don't (normally) move with their casters, and we also know that the bubbler leaving the bubble "pops" it. Moreover, both the Mistborn Adventure Game (a dubious source, but worth looking at) and the implications within the books suggest that the size of the bubble cannot be changed once it's been cast. So Wayne can't grow and shrink his bubbles however he pleases. The MAG would also have us believe that the "compression factor" of time bubbles—the factor by which they slow or speed time/movement—is set once the bubble is created. This is somewhat up in the air, though. This quote tells us that flaring the metal can give a greater compression factor. The question, then, is whether Wayne's bubble would have collapsed if he stopped flaring (going down to just a normal burn) or whether it's compression factor would simply have dropped. Myself I'm inclined to think that it's the latter. If not, though, then that tells us that bubblers can mess with their bubble's attributes, at least to some degree, even after casting them. I doubt this because of how "bubbler independent" these bubbles generally come across as, but it is a genuine possibility. Beyond being able to flare for increased compression factors, we have this quote about bubble size: Strength determines size This implies rather strongly that flaring could also be used to increase the bare size of bubbles as well as increasing compression factors. While originally I'd thought that there might be a tradeoff/inverse relationship between the size and compression factors of bubbles, the bubble size/strength WoB above looks to put the kibosh to that scheme. The possibility of misinterpretation/misunderstanding still leaves open the possibility of a super-small cadmium bubble being able to slow time to near stand-still or the like, but it's unlikely barring future, more expansive WoB. (x6) Effects of duralumin/nicrosil: Brandon has historically been suspiciously coy about what would happen when you use duralumin/nicrosil in combination with the external temporal metals. RAFO's are the order of the day. But we can extrapolate a bit from our above discussion of the extent to which bubblers can consciously determine the attributes of their bubbles. One thing to pay particular attention to in this context is how duralumin/nicrosil flares normally work: They compress all the power of the metal down into a split second. Not a singular point in time (because that's impossible and it would result in a hyper-steelpushing Vin exploding), but some very small interval during which the power rips out of the Allomancer all at once. Tied to this is the fact that burn rates are subjective to the bubbler, as discussed above. To get back to duralumin/nicrosil bursts, though, this information about subjective burning suggests quite strongly that a bursted-bubbler should only experience the normal "instant" of super-flaring. This means, for one, that we can't expect a nicrobursted Slider to get 10 hours on the inside to 100 on the outside or the like, since he'd have to be burning the metal for 10 hours subjective in that case. Besides running afoul of (reasonable, I think) interpretation of the subjectivity WoP, anything else would just be rather odd. Otherwise, we could end up with a time bubble that is no longer being sustained by the burning of metals, it seems, and the duration of which is oddly both too much in- and too much out-of the control of the Slider: he cannot stop burning bendalloy to cancel the bubble, yet he can still walk to the edge and "pop" it well before the bendalloy he's already burned has run its course. -Recall that, in the normal course of things, bubblers can just stop burning their metal at any time to drop the bubble. Not so in super-ultra-weird bursted bubbles that take a long time to run out subjective. - Assuming short subjective experiences, then, we have a few options to explore. In both cases, there are only two places all the extra energy from the duralumin/nicrosil flare can go to: increasing the compression factor or increasing the size of the bubble. Both are valid options, I would argue, because of what we've already seen about more powerful Allomancers making bigger bubbles, as well as flaring being able to increase compression factors. To avoid the "10 hour 'instant'" problem, any increase in compression factors must be... dramatic. For bendalloy, it'd have to be such that the Allomancer only experiences an exceptionally short interval. This means that you need to burn up all of the bendalloy in that short time and speed yourself up accordingly. This results in a situation where Wayne experiences 0.5s while the outside world experiences 0.00000000005s or the like. For cadmium, it'd be similar, though you can get a tad more utility out of it. Because the only time we have an upper limit on is the subjective experience of the bubbler, Marasi could experience 0.5s to 5000s on the outside, or the like. Alternatively, I think that we could maintain our usual compression factors (or at least something close to them) while pouring most of the extra energy into increasing bubble size. So you'd still get the usual 1:20 or whatever it is, but over a much larger area. Alternatively alternatively, the WoB about size not being inversely proportional to power could be generously interpreted to mean that you can expand both the size and the power of a bubble for the same flat rate, without any additional cost to do one as long as you're already doing the other. -- Theoretically bursting a bubble could be either an option between these two sets of results for the bubbler or Brandon could have it such that one of the results (either compression factor increase or size increase) is "locked in" and happens automatically when you burst. I am highly doubtful that it's the second case. We've seen Vin be able to have quite a bit of choice/nuance in her flare usage before, such as when she targets specific objects for steelpushes. If it is the second option, then I would wager (quite heavily) that we're locked into increasing sizes rather than compression factors. I don't know about you, but I find the idea of magically passing half a second while the rest of the world passes a millionth of a second to be... unimpressive. There's nearly zero utility for Sliders in the case where duralumin/bendalloy locks you into increasing compression factors. Besides my just saying "ugh that's not cool so it can't be true", I don't think Brandon would be so storming coy about the matter if the effect was this lame. So, in conclusion, I find it likely that using duralumin/nicrosil in conjunction with the external temporal metals results either in the bubblers being able to allocate the energy freely between increases compression factors and increased size or in the bubble being automatically expanded in size. (x7) Effects on various magics, aluminum: Time bubbles interfere with "almost all forms of investiture," it seems. Time bubbles interfere with investiture This makes sense from a Doylist perspective because things get weird and complicated very very quickly if you start talking about over-time steelpushes and the like. Realmatically... I suppose we can swing it easily enough, given that it's known how investiture tends to interfere with other investiture, and time bubbles are essentially a big fat mess of investiture altering the entirety of reality within their sphere (pseudo-intentional pun). It's possible that there's more to it than just investiture conflict, though... Some magics can bypass the interference of bubbles: Emotional allomancy works through bubbles because it's "over the top": This WoB is a bit unclear, but my interpretation here is that emotional allomancy operates on a Spiritual level, and in so-doing bypasses the timey-wimey effects of time bubbles. This is interesting because it implies that the way that time bubbles normally interfere with investiture is more a side-effect of their manipulation of time/space rather than a direct impact of the presence of the bubble. - We also we have this pseudo-RAFO on atium. Time bubbles likely affect atium in interesting ways Not much to see here, except the fact that it would be cool to see means that something likely happens besides just "oh the bubble interferes and you can't see anything outside of it." - Then there's aluminum. How exactly aluminum interacts with time bubbles is unknown, but it's generally a good bet that aluminum is going to do something screwy to just about any magic system, especially in Allomancy. We do have a bit on the matter. First, a RAFO: Aluminum RAFO Then a very confusing recording with an asker-approved paraphrasing: Aluminum creates dead space: (heavily paraphrased from a recording with approval by the question-asker, for clarity) If we're to take this WoB at face value, it looks like you could wrap yourself in aluminum foil and stroll through time bubbles without being affected by them. Which would be weird for essentially everyone involved, and suggests that aluminum bullets would also be unaffected. (x8) Faster than light (FTL) possibilities: Ah, FTL. The thing I tend to rant about the most. Brandon has stated multiple times that the third Mistborn trilogy will be a space opera where Scadrians have figured out how to get FTL using Allomancy/Feruchemy. Moreover, on later occasions (such as a WoB I'll be quoting shortly) he's narrowed it down to "Allomantic FTL". So we know that Allomancy is at the very least directly involved in enabling FTL, and it's quite likely that you can get FTL using only Allomancy and no Feruchemy. It remains to be seen whether the "mechallomancy" that they use on Southern Scadrial is at all involved, perhaps enabling bigger/more nuanced versions of known magical effects. --- Time bubbles are the natural place to look for FTL, then, because they change the nature of space-time, as many of the more plausible theoretical and sci-fi FTL-enablers do. The laws of physics in the cosmere are ours barring Spiritual shenanigans, so we still have to worry about relativity and can't rely on the infinite mechanical energy from Feruchemical iron or the like to get the job done. We have some quotes on the matter: Lost energy Several years later, Brandon PAFO'd the following question, leading to this eventual answer: Subjective burn rates How delightfully ambiguous of you, Peter. :\ Now Brandon had to initially PAFO the question, suggesting that this answer wasn't on the top of his mind and thus that it's not related to FTL. But there's also the possibility that yours truly simply managed to phrase it in a confusing manner. Brandon's initial PAFO was fairly fast, best characterized as "my give up" as the question was put to him verbally. Subjective burn rate PAFO: So it remains a possibility that these two answers are in fact linked as, as a first-blush look at their very similar diction suggests, and that this kind of subjective burn rate question is linked to FTL. Or not. Ambiguity. (x8.1) How I think FTL would work with time bubbles: I have multiple threads on this matter. The most recent (and only even possibly accurate one) can be found here. But I will summarize its points. Basically, you want to take advantage of how time and movement come uncoupled with time bubbles. You take a spaceship, accelerate it to some relatively fast but still reasonable speed (some very small fraction of the speed of light), and then do some shenanigans with time bubbles. What you want to do is encompass the entire ship in a bendalloy bubble which is anchored at a point moving, from your point of view, in a direction directly opposite that which you want to go. So if I'm moving from A to B, I want to start moving towards B and anchor my bubble on A. That'll get you accelerated movement. The ship will move at speed-relative-to-bubble-anchor * compression factor. Even if that gets you over the speed of light, the ship doesn't care because subjectively it's all good and not violating any physical laws or the like. You'll also need to encompass the ship in a cadmium bubble that, quite crucially, is stationary relative to the ship. This so that you can offset the extra time the ship/crew would normally experience in the bendalloy bubble. So now instead of the crew seeing the rest of the universe crawl by, they see it move by super fast. This works because the cadmium bubble shouldn't affect the movement of the ship through space at all (since it's anchored to the ship). Of course bendalloy bubbles are normally very small, so another thing I want to do is use nicrosil to increase their size for a second or so, just enough time for our ships to "teleport" (which is what this'll look like, essentially, from the outside) the length of the bubble. This being the effect of Allomantic nicrosil, as well as some ability to anchor bubbles at will and at different places depending on the user are both needed for this to work. - If Allomantic nicrosil fails us, we likely need to fall back to mechallomancy. So far as anchoring goes, I'm quite confident that out of all the options we have from up in the Anchoring portion of this post at least one of them will work out well enough. One example of anchoring that might work is the "cutting" WoB up above. Brandon was quite specific that the bubbles get their frame of reference from what they're cutting through: what the edge of the bubble is intersecting. So all we have to do is have the cadmium bubble intersect the ship and the bendalloy bubble intersect only the interstellar medium and we're golden. Some more thoughts on how the interstellar medium would perceive itself here, though most of our concerns on this score are actually overridden by a recent WoB stating definitively that (just about) all matter in the cosmere is at least a bit sentient, meaning that even little interstellar dust specks that no one's ever seen have some kind of Cognitive aspect: one that I would bet sees itself as "still" just like everything else in the universe acts like. - Here's Brandon's comment on the matter, insofar as I was able to ask him about it: FTL needs Nicrosil and/or time bubbles This is reassuring in that at least some part of my framework seems to be on track, but the "haven't figured it out yet" is troubling. In regards this "haven't figured it out yet" problem, we have a few other WoB's to look at. Missing big piece for FTL Quite awhile after this WoB, Aeromancer was kind enough to ask these questions: Unseen Allomancy required for FTL: Historically my hope here was that the missing Allomantic ability (that "very big important piece", it seems) was nothing more or less than something that enabled bubblers to anchor their bubbles in different locations. But given recent developments that suggest that it's fairly easy to mess with anchors, that might not be it. Another trick might just be using mechallomancy to manage bubble size/timing and the like, though the extent to which that represents "Allomantic abilities" is doubtful. (x8.2) Mechallomancy and time bubbles: As we saw in Bands of Mourning, mechallomancy plays very well with time bubbles. Marasi was finally able to live out her dream of trapping people in place in her slow-bubble as she laughed with glee, Wayne was able to toss his speed bubble around, etc. The question, then, is how this might interact with FTL. After all, what's a good sci-fi yarn without a redlined engine with exotic fuel, the sabotage of which leaves our intrepid crew stranded on a strange world? The immediate thought is the potential for different answers to the anchoring problem (might you be able to manufacture primer cubes that "think" they have one or another frame of reference?) or the potential to turbo-charge a cube for big/fast bubbles. Satsuoni, though, raised the interesting idea of just periodically tossing a bendalloy-primed cube out ahead of your ship as a way of getting a not-anchored bubble to travel through. Pick it up after you pass by, rinse, repeat. We have some more recent WoBs also suggesting at mechallomancy as a component of FTL. It's still unclear whether it's strictly necessary or just useful for practical applications (i.e., if you could zip along a life-boat sized ship with a team of perfectly trained Allomancers, but to do anything bigger/better you need some machine-precision/scale). Merger of magic and technology for FTL: Contemporary trilogy tech hints at FTL: (x8.3) Disclaimer: I would also like to note that I am not the only thinker in this field. Others have proposed theories talking primarily of using bubbles to achieve Alcubierre-style effects, and there are a few more out-there theories. Myself, I suppose an Alcubierre drive could certainly do the trick, it's just that my level of physics-brain, as well as my understanding of time bubbles, doesn't seem to imply that that's something you can get with time bubbles as we know them. ---- Fair warning that this section is by far the most "Kurkistan is a narcissistic monster" part of this thread—a thread which is already devoted to essentially saying "now listen up. I know everything so sit down and listen", so raising the level of egocentrism is an impressive feat. If you disagree with my conclusions here, then I wouldn't say that you're just going against the fundamentals of how time bubbles work. For the rest of the thread, though, I must say that I've yet to see any other cogent and plausible analysis of all the details. If only because I immediately jump at anyone who tries to develop one and spike-out their knowledge. (x9) Realmatics: Talk of Realmatics is for another thread, I think. I still have yet to really dig my teeth into this aspect of time bubbles (which is somewhat ironic coming from me), so most anything I put here would be new theorizing, not tested by time and thought/criticism as just about everything else in this thread has been. So, in short, we know that whether or not you're "in" a bubble is governed by the Cognitive on some level. We know that there's likely something Spiritual going on with how bubbles get "anchored," not least because "connectiony" things are nearly always Spiritual. There's also fairly good evidence that the movement-altering effects of time bubbles are "real" in the sense that sped-up objects actually have extra kinetic energy, rather than just sliding about due to magical shenanigans without any change in their intrinsic inertia, given Peter's comments on the matter. Whether this means that the change is Physical is not 100% clear, though. Most else, I think, is speculation that someone (probably me, given historical precedent) will get to at another time. (xCn) Conclusion: So thank you for your time. That's about all I can think of off the top of my head. I'll incorporate anything I missed if I think of it or someone brings it to my attention. In particular, please feel free to point out any parts of the original draft that I modified/excised inappropriately, if they catch your eye. If you came here because I threw a link at you in some random thread: Did I answer your question? (xAr): WoB Archive: For archiving purposes, here are all time bubble- and (possibly) FTL-related WoB's and WoP's I know of/can remember. Not all of these are referenced in the main body of the post. Change List: Re-wrote into version 2.0 after SoS and BoM - 1/09/2017 Added WoB on time bubbles affecting time - 2/11/2017
  5. Bands of Mourning and Shadows of Self SPOILERS ahoy. I'm leaving the thread here because it really has nothing to do with either of those books beyond the evidence they provide, but this is more of a "General Cosmere Theory: Mistborn edition" than a proper Mistborn-book thread. We've had a swathe of new time bubble information with Shadows of Self, Bands of Mourning, and WoBs from the last few tours. We're to the point where there's some genuinely new/interesting information, so it would be inappropriate for me to just shove the first them I think of into the giant mega thread of death and call it gospel. So here I'll post all the new WoBs and brief overviews of new in-book information and we can discuss implications, etc. WoBs: Nested bubble size interaction: Aluminum creates dead space: (heavily paraphrased from a recording with approval by the question-asker, for clarity) Accelerated aging (paraphrase): Watches need to be reset: Doylist rationale behind bubble mechanics: (emphasis added) Emotional allomancy works through bubbles because it's "over the top": Included as soon as you touch: Cancelling only in area of overlap: Nicrosil/Duralumin RAFO: Conservation of momentum on entrance tied to redshift-solution: Cadmium hermit can time-capsule self: Bubble size/strength more controllable than shown, size not inverse to strength: Moving bubble's effect on intersected object: -Kurk's Kommentary: Well that's interesting. I'd been assuming relativity of reference frames, but it looks like there needs to be some other mechanism at work to decide how the cork is "really" moving. I don't think this necessarily messes with current FTL models, but it requires some rethinking. The "everything we know" thread will need to be reworked to excise my relativity-based analysis and incorporate this. --Also this thread is wrong on more than one level. Book evidence: Jostling/deflection/refraction of objects traversing bubble borders In Shadows of Self when Marasi foils the assassination attempt on the mayor, it's noted that a bullet fired out of the cadmium bubble deflects on its way out. Then in Bands of Mourning there's some exposition while Marasi and Wayne are hiding in the grave: bullets entering the bubble get deflected as well. The words used are "anytime something entered a speed bubble, it was refracted...": recall that I'm weird, and that all time bubbles are referred to as "speed bubbles" in-world. At this point we've got definite in-text evidence of bullets being deflected on leaving bendalloy and cadmium bubbles, in addition to them being deflected on entering bendalloy bubbles. At this point I think we're pretty safe in saying that all transitions over bubble borders produce jostling. -I didn't spot any other mentions of in/out deflection in the two books, but and very open to being corrected. We also get the implication that shooting into cadmium bubbles causes deflection. This implies that the bubble being up prevents people shooting you: so either cadmium bubbles have some hitherto unknown bullet-stopping ability or the property the emphasized part references is just deflection. Moving bubbles: The fight on the train in BoM has a train-anchored bubble. Cool. The bubble even "captures" a cork that Wax throws at it (the implications of mechanics of this hashed out over an over-long WoB/P up above). This "heayy and fast" is quite fascinating, and runs a bit counter to the "cutting" WoB on what defines bubble frame of reference. It's reasonable enough, though: after all the bubble is based on "frame of reference for the cognitive things around" the bubble, so if you're a big heavy fast thing I'd guess that you have a more solid argument/perception of what "still" is then something small and slow and flimsy. - It's implied in the scene where the gang is testing out the primer cube by tossing it around that the cube spits out a bendalloy bubble while its in mid-air, then proceeds to land on the ground near Marasi with the bubble still centered on it. This would imply that the cube either has some special ability to anchor bubble frames of reference or (as I'm about to suggest) that while the bubble was airborne it simply "defaulted" to being anchored to the cube, but when it landed it assumed a normal ground-based frame of reference. This supported by Marasi walking up and picking up the cube off the ground without any mention of the bubble shifting. ------ Nothing else bookey comes to mind immediately, beyond, obviously, the interaction of time bubbles with the primer cube and mechallomancy in general, as well as the implications that has for FTL. Satsoni, for instance, has raised the idea of just tossing the cube out ahead of a space ship as a way of getting a not-anchored bubble to travel through. Speaking of primer cubes: It occurs to me that bubbles must necessarily have some kind of "default frame" they assume upon first creation that's then maintained by the bubble's surroundings. This reconciles the ability for flung bubbles, airborne allomancers, etc. to have stable bubbles with Brandon's statements on frame of reference being determined by what the bubble "cuts" through. Actual analysis of all of this to come, but for now I just wanted to get everything down on paper. Discuss away.
  6. Get comfortable, folks. This is the kind of theory thread that I fully intend to save an off-web version of so that I can pull it off of my brain-drive 20 years from now when Brandon writes the third Mistborn trilogy. I do not intend to see any left-out sections at that point in time. Also, slight AoL spoilers. Background: Evidence: Assumptions: Main Theory Note: I'm scavenging this from several of my posts on the other thread, and will try as best as possible to credit others where appropriate. Feel free to call me out if I miss someone. Goal: Use cadmium bubbles to achieve infinite free acceleration and bendalloy bubbles to achieve FTL travel through real-space while retaining less than 'c' speed from the perspective of the ship. Propulsion: If we can actually break c in real-space in the Cosmere, then this will get you there. Otherwise, you're going to need. . . Time-Fiddling Movement: Ship Design: EDIT: Quick update: This is all wrong. Objects are either totally in or totally out of time bubbles based on their Cognitive aspects and frame-of-reference shenanigans means that anything matching the time bubble's vector will not be accelerated by it.
  7. I apologize in advance for putting you guys through this again. I was going to post this as a reply to Windy's new Q&A thread, but it got a bit long and I didn't want to hijack the topic. Some background: Don't bother reading this. At the end of the day, I discover (in a very tail-eating, rambling fashion) that frame of reference is very irritating and means that any time bubble "moving" with a ship would not affect it's motion through space. That's what matters now. Windrunner's new thread also gave us some new information on FTL: I have some thoughts on this snippet here, but for now I'll just be posting my current theory of FTL in its entirety on paper. I may have said this before, but it's what's most plausible to me given our current understanding and one or two crucial assumptions (hopefully the "big piece"): Give our aforementioned frame of reference problems, large-scale "teleporting FTL" is all that I can think of at this moment. This requires a pair of assumptions to be manageable: bubble "anchors" can be set relatively fluidly and nicrosil/duralumin increase the size of at least bendalloy bubbles to some fairly large extent. Given these requirements, an FTL ship would need at least one bendalloy/nicrosil combo (or a mistborn or Hemalurgist or whatever: it doesn't matter) and one cadmium misting. How it works: First, accelerate the ship to as close to light speed as you can manage. There are a variety of ways you can do this--infinite energy from Skimmers, very violent Pushes with Coinshots on anchors, etc. Then do a series of "jumps" until you get near your destination, at which point you start slowing down. Procedure to "jump": For each "jump" of the trip proper, have the cadmium misting puts up a bubble that encompasses the entire ship so that the crew and ship's systems don't need to experience any of the "extra" time during transit. Then have the bendalloy misting super-bubble the ship with the aid of duralumin or a nicrosil misting. Hopefully this bubble is several kilometers, if not several hundred, thousand, or million kilometers across. From the perspective of Scadrial, the ship will traverse this region of space at several times the speed of light: the (cadmium-bubbled) crew has the same perception, after you factor in normal relativity for the ships "real" speed. To help things along with this absurdly large bubble, empty space could be far easier to bubble (as I suggested here) because there is less stuff to be affected in near-vacuum, so an improved Bendalloy bubble, instead of just being 5 times bigger or something as it would normally be, explodes once it gets beyond the confines of the ship. This all sounds plausible enough, but the real problem lies in frames of reference of the "bubblers:" The bendalloy misting needs to set his bubble's frame of reference to the totality of matter in local space: surrounding stars and galaxies, primarily. This will get us a good enough "motionless" for our purposes, and allow the bubble to be "still" in space relative to our near-light-speed ship. We're okay with not having any absolute "rest" frame of reference because the entire point of FTL is to traverse local space. If you want to be a bit more parsimonious, you could just set some local star or other as your frame of reference, and it should probably work too. The cadmium misting, on the other hand, needs to set her bubble's frame of reference to be motionless relative to the ship itself, and hopefully set to approximately the same (if not an even higher) "compression factor" as compared to the bendalloy misting's bubble. This way, the crew and ship won't experience centuries of travel subjectively. Bubbles cancel each other out (multiplicatively), so this could work, allowing the cadmium-bubbled ship to move through the space contained within a much larger bendalloy bubble. The crew-saving cadmium bubble won't slow the ship down because bubbles that are "at rest" relative to anything within them do not have any affect on that object's motion through space. The end result is that you get "teleportation" where the S.S. Vindicator moves through normal space in a series of quick bursts many times faster than the speed of light, like when Wayne uses his speed bubbles to "teleport" in AoL. If you get multiple bendalloy and cadmium mistings together, you can exponentially increase how fast these bursts are without any ill-effects. Just to reiterate, this makes key assumptions: 1) The ability to not only "decide" where to anchor a time bubble, but to have two adjacent mistings choose different targets. 2) The effects of duralumin/nicrosil on bendalloy mistings being such as to increase the size of their bubbles. -2.5) The emptiness of space increasing the size of time bubbles as they have fewer objects to affect. ----- And so that's that. I've said essentially the same thing before, if I recall correctly, but not so completely. Given that Brandon has said we still don't know "a very big important piece of the puzzle", I'm not exactly flush with confidence that my theory is right. There remains the slight chance that assumptions 1, 2, and possibly 2.5 are these pieces of the puzzle, though.
  8. So I was writing up my absurdly large time bubble thread (what else is new?) and I got the section about bubble occupancy. For the first time, I actually sat down and thought about the implications of a particular new WoB in light of some fairly old knowledge: Namely, how do we reconcile the facts that time bubble borders are static (i.e., don't distend ever) and any living thing touching a time bubble is affected by it with the fact that passengers in a bubbled train that doesn't notice the bubble won't notice it either? The WoBs: Living touch enough Bubble borders static VS. Bubble occupancy ----- How to reconcile these, then? It seems that the key to this question is to ask how time bubbles "find out" whether or not objects should be included in their effect. We see that objects are included or excluded as wholes: You won't be speeding up one leg of a chair and leaving the rest untouched, for instance. Yet objects do not lose their individual identities when they become a part of a greater whole. We saw in WoR that parts of the whole can both have their own beads in Shadesmar (their own individual Cognitive aspects) and be represented within the beads of their parent (and presumably their parent's parent and so on). The doors and lamps in Kholinar's palace have their own distinct beads at the same time as those same objects are a part of the "PALACE" mega-bead. We see, then, that time bubbles know not to include the child without consulting the parent. You won't see the chair leg being included on its own because the bubble somehow knows that that leg has a parent aspect and that that aspect is not included. The bubble then doesn't include the leg. Where is this answer derived, though? 1. Does the bubble look first at the leg, then see it has a parent, then look to the parent, then check if it has a parent, then see it doesn't and determine if it would like to go fast today? -This requires some mechanism of "trickle down" for all the child objects to be suitably affected/not-affected. 2. Does the bubble simultaneously survey all potentially-included objects, figure out the tree, and then only bother to actually look to the root nodes for occupancy? 3. Does the bubble simply ask the leg "would you like to go fast?" and then the leg, as black-box, replies Y/N after querying its parent? I am somewhat disposed towards the third option, myself. It's a bit of extra work on the part of individual aspects, but requires fewer/slower fundamental changes than the first and far less unbelievable work than the second. The first option requires that pesky trickle-down mechanism. It requires that either the bubble remember everything it's looked at and associate them with their parents as it finds them or that the parents be able to tell all their children its "decision". The second requires a large degree of "thought" on the part of the bubble: it takes place in two distinct steps, with some concept of consideration/evaluation in play between the steps. The bubble somehow "looks" at everything before "actually" looking at only a subset of those things for its actual answer. It seems overdone. The third option does demand some degree of work from individual objects. It requires that random peice of wood #7 go through the trouble of actively figuring out whether it and/or its parents want to be included in the bubble. This could go on for dozens or hundreds of layers, depending on how fine-grained we get. -- Another thing we must consider is how what objects are included in the bubble is decided. Yes, there's Cognitive "is this thing 'in' this space?" questions, but who's asking and who's answering? Are individual objects returning their assessment of whether they're within the bubble's space, or are third parties, or is the bubble itself determining this? The answer to this question has special weight for Option 3 above because that option doesn't allow the bubble any input into whether the root object is included. -- Getting back to my own judgments, I would also say that the objects' Cognitive aspects in the broad sense (so including both personal and external opinion) determine "where" they are in space, and then that object reports whether it thinks it's "in" the bubble if some part of it is straddling the line. I don't think it appropriate to saddle a distortion in space-time with too much thinking. This presumed lack of cognitive ability is yet another reason to discard Option 2, also. I think it fair to proceed as if we've narrowed the reasonable options down to 1 and 3, then. Not being sure between these two still leaves ambiguity as to exactly what these Cognitive aspects look like, though. In the case of Option 3, then the only thing we need in order to reconcile the WoB's is for child objects to know who their parents are. You can have a bunch of illegitimate passengers running around saying "this train is my parent" without the train ever being aware they even exist, let alone that it allegedly has a few hundred children. -- But in the case of Option 1, things are nastier. We've already essentially excised the time bubble from any computational heavy-lifting, so requiring that it remember/associate children with parents seems untoward. That leaves the parents being able to inform their children of what's what. How, then? I see two fundamental ways. Either the parent knows all of its children and tells them all personally or it "broadcasts" some message to all saying essentially "My kids: listen as I say...". The broadcast idea is rather uncivilized, and ghastly-expensive/loud. If the parent has no idea whatsoever who its children are, then it has to tell everything it can reach the message. And the kids have to all be listening for it and be able to actively distinguish when their parent has a message. Ugly and I don't see Brandon designing it that way, frankly. On more Realmatic grounds, the Cognitive isn't really that kind of place. Nor is the Spiritual, when you get right down to it. We've yet to see any simple broadband "tell everyone the news" communication Realmatically. The Spiritual has sets of 1:1 connections while the Cognitive seems composed of a bunch of quite singular entities that, to their core, know what they are. While this "knowing what you are" could quite easily extend to "just knowing" your parents or children (and thus perhaps going on to forge and maintain Spiritual bonds with them for communication), it doesn't seem to extend to knowing and being able to communicate with your local area. So the other way is that the parent knows all of its kids and tells them personally. How kind. But the problem with this is the ephemerality of some of the relations we're talking about, and how quickly they'd have to be established and torn down. When a passenger sets foot on a train, this model requires that that train immediately become aware of that passenger as its child and see itself as containing him. That's the impact of this set of WoB's. And that's rather absurd on the face of it. Besides the "absurd on the face of it" thing, which isn't exactly a scientific assessment, we also have good reason to disbelieve such ephemerality: Soulcasting. When Shallan soulcast the ship in WoR, she did not, as it turns out, turn all of its human occupants to water. Despite this, I think it fair to claim that those same occupants would have been unaffected by a passing time bubble that the ship ignored. So there are in fact at least these two levels of Cognitive identity to deal with: one where ephemeral occupancy does protect you from time bubbles but does not entail being "part of" something (the ship's occupants) on a deeper level, and another where long-term occupancy both protects from time bubbles and entails "partness" (the ship's various planks and whatnot). Following from this, the natural conclusion to make, I think, is that what separates these two types of belonging is exactly where children objects are accounted for in their parents' cognitive aspects. Perhaps some other division could be made, or we could make allowances for the difference being due to different degrees in which the parent regards the child, but such seems unwieldy and quite unnecessary in the face of a simpler option. -As an aside, I'd say that Soulcasting directly affects exactly those parts of an object which its Cognitive aspect (its bead) says are a part of it. So Soulcasting directly locates and transforms all those basic-level, non-parent objects which are a part of the whole according to the whole. The whole knows what all of these objects are, and is likely Spiritually connected to them, so I'd wager it easy enough for a Soulcaster to follow the threads down. ---- It seems, then, that we've discarded Option 1 in its entirety. That leaves Option 3, which, to recall, is the one where individual objects are queried by the bubble and make black-box replies which they base on querying their parents. This model requires only singly-linked lists rather than the more expensive doubly-linked tree option 1 needed. Any given object needs only to know either what object its parent is or that it does not, in fact, have a parent. In the first case it asks the parent whether they're in the bubble, in the second it makes the judgment itself. The parent doesn't need to remember any of this, only bothering to connect to its children for a moment as it replies to their individual queries. To the parent in this model, the child asking after occupancy is asked and answered in the same exact way as the bubble asking after occupancy. ---- So, to summarize, this leaves passengers in the train not "touching" the bubble proper because they don't really "interact" with it directly. At no point are they actually doing any work in assessing whether or not they're in the bubble: all they do is ask the train to make the call. When the passenger is in the train, he doesn't actually know whether or not he's touching the bubble because the only way he has to assess the question is asking his parent the train what the answer is. When the passenger is outside the train, though, the buck stops with the human and he is, in fact, "touching" the bubble with no Cognitive intermediary. Honestly I preferred distension to all of this, but given the WoB we have and assuming that I'm not drastically misinterpreting it (which I'm fairly sure I'm not), this seems the answer. ------------- Implications: This is likely the exact same way that clothing and/or held items behave for time bubbles. While I have my whole thread on the matter, I have yet to address what happens to clothing when people touch time bubbles. I think it intuitive and natural that their clothing is included in the speedup/slowdown as well. It wouldn't do to have a "pulled out of his socks" situation it a man poked a time bubble with his finger while running by. So perhaps the mans clothing is included in the direct, "this is part of the human's Cognitive aspect" sense. It's a bit more of an open question whether held-items (like weapons) would have the same privilege. To expand why such short-term "partness" is plausible, I would argue that such possessions (including held items?) are included (and eventually discarded) so easily because living beings are living beings. We cannot expect a train or a ship to actively track and/or have opinions on what's properly part of it: That kind of thing is decided entirely by the input/views of living beings who can make such judgments. By that measure, though, it could well be that the living things have much more malleable "part of me" parts of their Cognitive selves, and can in fact very quickly come to regard clothing/held-items as a part of themselves. In this human case, then, clothing and possibly held objects are included in the effect of the bubble directly because of the uniquely malleable nature of the living Cognitive aspect. -- Another point to make is that I would think a man wearing gloves who touched the edge of a bubble with only the gloves might get included, as the gloves are a part of him. That one's a tad in the air, though, as to whether the "part of me" extends so deeply. We wouldn't expect someone whose shirt got cut by a Shardblade to feel pained by it, after all. ---- Another expansionary point is that this might actually address the "man runs into pole at 500 mph" point I touch on in a few of my time bubble threads. The basic problem is that of a moving person who is trailing along a train (matching its speed) and then gets time-bubbled at 20x speed into the back of a train that is ignoring the time bubble. Does this man suffer damage from this collision? This "parenting" theory may solve this problem: potentially, the moment the man contacts the train his overriding "wait a minute, I'm now in contact with an object big enough that I consider myself to be 'on' it" is such that he begins to take his time-bubble-related ques from the train rather than his own body. So the man ends up painlessly coming to rest against the back of the train with no tangible impact, rather than going kersplat. ------ So... yeah. This is what happens when I get in time bubble mode.
  9. Critics Rave: "That's a very interesting way to look at it. The theory isn't all there, but it's thinking along the right lines." -BRANDON SANDERSON "a surprisingly coherent theory considering everything we don't know" -dj26792 "while I don't completely agree with the terminology you say, the ideas here are extremely extremely good." -Chaos "it's a great theory that makes sense with the Cosmere so far" -Straff Venture (before being cut in half, presumably) "supercalifragilisticexpialidocious" -Observer "good theory! Well presented and excellent concept, at least." -Odiums_Shard Table of Contents: Main theory (this post) Awakening --Start of the Greate War of Nepene (~25,000 words. You have been warned) --End of the Great War of Nepene ---New Orleans of the Great War of Nepene (Hint: I'm Jackson) Start of discussion about: -Cognitive/Spiritual interaction -Spren -Hemalurgy and Memory/Intelligence To Be Discussed in Depth: -Mostly Cognitive beings -Spren -Hemalurgy, specifically concerned with Memory and Intelligence theft -The relationship between Spiritual energy and Cognitive complexity -Predictions for how Forging works. -Senses of Awakened objects Related Theories: -Forms: --Forms: Simplified --Spren as Forms --Nature of Forms & Forgery ---Spiritual Realm as a Network of Connections -Realmatic Theory, Satsuoni edition --A bit divergent, but mostly on the same tack as mine. ---- Summary of theory so you can "get it" without reading the equivalent of a paper. -Lacks reasoning, evidence, and length. The Spiritual realm is the source of all motivation within the Cosmere, from gravity to punching someone in the face. It may or may not also provide the energy to enable these actions. Every object has it's own unique sDNA which specifies how it interacts with other objects and the Cosmere at large, once again from gravity to the sDNA of a Thug allowing him/her to burn pewter. Magic depends on this sDNA and can, on occasion, change it. The Cognitive realm (the least well-defined of the three so far under my theory) is where all of the computations and permutations of spiritual rules are considered--in the context of the layout of the Physical realm--on a moment-by-moment basis. It is, quite literally, where the cogitation takes place. Alterations to the Cognitive realm can alter how objects react to certain Spiritual directives, but not the directives themselves. It may, however, also serve as a natural interface between the Spiritual and Physical realms, allowing one to indirectly affect the other. The Physical realm is where everything actually happens, changes coming about in it as an end result of Spiritual directives being interpreted by the Cognitive realm. The results of those directives then affect the other two realms in turn and go on to determine what happens in the next "cycle." Main Block (Ridiculously Long): EDIT: Actually, my explanation for time-bubbles is far too simplistic. It may be that there are some rather sophisticated methods for determining what's in and what's out of a time bubble, and we have to account for frame of reference. Ugh. I suppose the "go faster" variable could be altered as a function of the frame of reference of the anchor, with an additional "time passes this fast" variable which is altered the same way no matter what (except for relativistic effects, perhaps. . .). EDIT 2: According to a Harmony quote, the Spiritual realm definitely provides the energy. EDIT 3: Unearthed another quote, this one from Elantris. A bit suspect, due to theological underpinnings, but the first concrete "the Dor/Spiritual motivates stuff" quote I've seen. So there's the theory. Read, gush over, criticize, analyze, substantiate, annihilate, and permutate to your hearts' content.
  10. I was going to append this to my Yet another FTL Theory thread, but it got a bit involved so I decided to make a new topic for it. Update: More involved even than I thought it would be. Sorry. I have some concerns about "Who bubbles the bubbler?"--okay, not really, but I couldn't resist. More accurately, what happens to the bubble of a Pulser or Slider under the effects of another time bubble? So: How is a Bubbler's burning affected, if at all, by being encompassed by another time bubble? Scenario: Wayne is inside the range of Marasi's bubble, and he throws up one of his own. Assuming the compression factors to be equal, Wayne will experience 1 minutes in his bubble for every minute passing in real time. So he'll burn however much Bendalloy he normally burns to sustain a bubble of X size with Y compression factor for 1 minute subjective. The key factor is the subjective experience of time, I think: The MAG (<sarcasm> that wellspring of entirely accurate Realmatic knowledge </sarcasm> [but really, the MAG is great. Just not accurate on some rather critical parts) says you burn Bendalloy at 5 minutes per charge subjective, and this also makes sense intuitively. But what if we put Marasi inside Wayne's speed bubble instead of the other way around? So time passes "normally" for those two while an area around the bubble is Pulsed. That's all well and good, but how is that exterior bubble being sustained? Cadmium within Bendalloy: Is Marasi burning less Cadmium than usual, since each second of her burning corresponds to many seconds when she would be burning if not affected by a speed bubble? If so, then what about the area corresponding to Wayne's speed bubble? That has to be sustained at a normal rate if she wants to properly cancel his bubble out and gain a "normal" passage of time, it would seem. But then we get a weird variable bubble-within-a-bubble-within-a-bubble effect, with the area that happens to correspond to Wayne's bubble getting special treatment. I think, then, that encompassing Marasi in a speed bubble ought to have some rather dramatic effects. I would say that her burn rate stays constant--which it really should, when you think about it, rather than being lowered against her will--and her slow bubble is getting a ton of extra energy: tens of times more per second than usual, at the least. It is as if her burn rate has been multiplied by whatever the compression factor of Wayne's bubble is, a super-flare just short of the Duralumin. Where to put this energy, though? As far as the part of her bubble within Wayne's bubble is concerned, everything is going normally. But the parts outside are getting an unexpected boost in energy, and need to put it somewhere. Increased compression: It could go into increasing the compression factor or Marasi's bubble. If so, Wayne's bubble would behave it was simply under the effects of one of Marasi's normal bubble, while the rest of the Cadmium bubble would be going crazy-slow. If so, then Marasi's time bubble would nearly smother Wayne's, with the the region encompassed by Marasi's entire time bubble moving at a the pace of a glacier with a bead of water moving alongside it, compared to the bullet of the rest of the world. If the normal Cadmium bubble gives you 100s on the outside for every 1s inside, then this one will give 10,000s outside for every one inside, while retaining the 100:1 ratio within Wayne's bubble. If Marasi had enough Cadmium to make 10,000s pass outside in 100s inside, then this bubble will last for the full 10,000 seconds in real time. Marasi and Wayne will experience 100 seconds, while those outside of Wayne's speed bubble will experience 1. Problem: This doesn't account for power gain. Assuming that Marasi is maintaining a constant burn, subjective, we're getting an absurd amount of time-distortion for free. This is because the increased compression factor of the slow bubble compounds back onto Marasi within Wayne's bubble, redoubling its longevity. Increased size: So we go to the other direction the energy from Marasi's "Bendalloy burst" can go: out. I would suggest that the size of Marasi's bubble would increase, while its compression factor remained constant. So Marasi experiences 1 second for every 100 seconds experienced within the rest of her bubble, as usual, but this 1 second is also experienced in real time. So her bubble dies out 100 times faster than usual, and all the extra energy goes towards increasing its size, encompassing and slowing more objects. I'd say the second option is nearly necessary, at this point, given the alternative. Do any other possibilities spring to mind for anyone? Bendalloy within Cadmium: And now we get to where I was trying to get for the FTL thread. I do hope you understand why I thought this needed its own thread What if Wayne was off to the side at the perimeter of Marasi's slow bubble: encompassed himself, but with about half of his speed bubble protruding. What happens to his bubble? The parts inside Marasi's bubble are getting their normal flow of energy, so they ought to be fine. But the parts outside are getting 1/100th what they expect. Disparate compression factors still don't make sense, so it seems we're left to shrinking the real-time portion of the bubble to cover 1/100 the area it normally does. But what if Wayne was standing on the outside of Marasi's bubble's perimeter? In that case, the half inside the slow bubble would be getting 100 times more energy than usual. So what does it do? Balloon out to 100 times its normal volume, it would seem. All of this is well and good, but things can get... weird. Weird Scenarios: (WTLR?! FTL? Here!? HOW!!!!!???) EDIT: Tell a lie! I got a bit ahead of myself. Overlapping regions that don't touch the bubbler should just cancel out, ala canon. Duh. Although... maybe they "cancel out" because of bubble-extension, to a certain extent, or bubble-extension is still a factor. I'll leave the text here, because I wrote it and it may still be applicable, but it's by no means as sure as I erroneously thought a few minutes ago. EDIT 2: I suppose the cancellation is just a 1v1 battle for space-time-warping supremacy. In that case, the time distortion is an effect on an area, rather than the objects within that area... But that doesn't mesh with the Cognitive nature of time-bubble occupancy, really, since bubbles seem to work on the level of objects. This does require some thought. --- Scenario: Wayne outside Marasi's bubble, at the perimeter. Marasi has walked to near the perimeter of that self-same bubble, but is on the inside and out of range of Wayne's normal speed bubble. Wayne casts his bubble. The half in the bubble expands drastically. Marasi is encompassed, now experiencing 1s/s--or she would be, if this didn't increase the size of her slow bubble. So Wayne is now encompassed by Marasi's slow bubble. His energy delivery to his own speed bubble normalizes, shrinking it back down to normal. So now Marasi is outside of its effects. So her bubble shrinks too. So now Wayne is outside her bubble's effects, so the half of his bubble inside Marasi's slow bubble expands. So Marasi is encompassed... Yeah. And this all might happen in a split second each time, if not instantaneously. I suppose a slower-than-instant expansion/collapse of the bubbles would allow for a stable oscillation between states, but still, this isn't a good thing. .... Or this stable oscillation between states just unlocked the real secret to FTL. Meaningfully, in such a scenario, we have a constantly expanding and collapsing Cadmium bubble. Expanding by tens of times in volume, mind you. Might do some funky things to space-time, I'd say. Maybe a way to actually achieve a warp field? We've now reached beyond the scope of my (pseudo-) competency. Physics dudes? ---- We still have nested Slider/Slider and Cadmium/Cadmium to consider... Maybe later. This is getting long enough as is. --- Aw, why not. It should be fast. Bendalloy within Bendalloy: Bubble of bubbled misting(s) expands. Could get oscillation with this as well, I believe. Cadmium within Cadmium: Bubble of bubbed misting(s) shrinks. Also a chance of oscillation. Wow, long. Sorry. Really. Any thoughts, anyone? EDIT 3: Oh yeah, I kind of forgot the reason I'd started all this. In my defense, it is an involved and fascinating topic. So in my FTL thread, I have a ship within a Cadmium bubble within a duralumin/nicrosil-induced massive Bendalloy bubble. So both bubblers are encompassed by the other's bubble. So by the logic I've outlined already, the Cadmium bubble should expand--a lot--when the Pulser is hit by the Bendalloy bubble. The Bendalloy bubble should then proceed to shrivel when it hits any area outside the influence of the Cadmium bubble, since it's getting less energy than it needs. So the full size and effectiveness of the Bendalloy bubble is only realized when it's in contact with the Cadmium bubble. I guess we're lucky the Cadmium bubble is traveling with the ship, then. I think we're still good.
  11. Back when Shardlet asked those questions about time bubbles, I said that "this means that bubbles are almost certainly a straight effect on local space-time, rather than an effect on local objects that happens to be roughly spherical in shape." gnimhey was kind enough to point out that that is a very stupid thing to say (in much more diplomatic terms). I started typing up a reply to him, then started thinking about some other time bubble related stuff, and then... Well, you guys know me well enough by now to guess what happened next. Thread: My comment that time bubbles were "a straight effect on local space-time" was specifically getting at an earlier theory of mine where I suggested that time bubbles weren't actually a spherical area of effect, but instead simply an incidentally spherical manifestation of the limits a bubble reaches when it "runs out of the power" making everything around it experience time differently. The fact that the presence and absence of air particles has no impact on how "far" bubbles can reach almost certainly throws such a model out the window, and that was what I was attempting to emphasize. (And yes, I do recognize the irony that the founding post for my overarching theory of everything came to a fundamentally incorrect conclusion on the topic that I had been trying to address from the beginning.) Fuller Explanation: Alternatively, a more nuanced theory can still make bubbles fundamentally spherical, but say that there size is a function of how much "energy" they have to put into altering objects they affect. So a Slider can cast a bigger bubble if it has fewer objects to worry about than if there are a lot. Under that model, there should be a measurable difference in the maximum size of a bendalloy bubble cast around a pool table full of billiard balls versus that same table when its empty. That second theory also goes out the window, since the atmosphere is the fullest pool table you can imagine, chock full of pool balls bouncing around and whatnot. If you can get rid of it and have absolutely zero effect on the size of a time bubble, then that casts near mortal doubt onto any theory that bases the size of time bubbles on how much "energy" they have to put into the objects they affect. -- Something energetic is still happening with encompassed objects, though. It's just that the bubble doesn't seem to care at all about the costs of that energy. It follows, then, that time bubbles don't directly affect the objects they contain, but instead affect areas of space-time (here I quite explicitly mean to include that area's Cognitive and Spiritual components), with the effects on objects trickling down. --- NOTE: It occurs to me at this very moment that the maximum compression factor (how much it alters the flow of time) of the time bubble could still be affected by the objects it contains. We know that flaring a metal can get you a higher factor, but not necessarily a larger bubble. I've been assuming the energy costs of the two to be somehow proportionate, or at least that larger bubbles cost more energy, but I could be wrong. I'll keep acting like making a bigger bubble is of the same "cost-type" as raising that bubble's compression factor, because I think it reasonable, but that's not a certainty. ---- Regardless, I phrased myself poorly and misleadingly, and now gnimhey has gone and given me an excuse to discuss the remaining possibilities for how time bubbles work, given our current evidence. Time Bubble Mechanics (not Realmatically, but just on a brute-force Physical level): First, some facts: Time bubble overlap is multiplicative. Time bubbles overlap like Venn diagrams, they don't just "cancel" one another. Atmosphere has no effect on bubble diameter. Aside: Time bubbles accelerate or slow objects as a function of some frame of reference, independent of how they influence the passage of time for that object. -Just my own reasoning, not WoB. It's solid reasoning, though, and necessary once you think of it. --- I infer from these facts about time bubbles that they directly impact an area of space-time (once again including all the Realms), rather than the objects within that area of space-time. Otherwise we would get exploding bubbles when they overlapped, and overlap would really have to be additive rather than multiplicative. Not to mention all my analysis on what the atmospheric answer tells us. Specifically, I would posit that time bubbles essentially alter space-time in the Cosmere such that: Time passes at t*x speed for objects in this area of space; and these objects move at v*x velocity along their vectors relative to <the bubble's frame of reference>. :With 't' defined as the rate of the passage of time before the bubble's existence. :And 'x' as the bubble's compression factor. -Either x>1 or x<1 depending on whether it's a Bendalloy or Cadmium bubble, respectively. This rather neatly tells us how an area reacts to both "competing" time bubbles (Bendalloy vs. Cadmium) and "cooperating" bubbles (B+B or C+C). Just multiply whatever 't' you find (1 if the area of space is normal, 1*x if one other bubble is affecting the area, 1*x*y if there are 2...) by your bubble's compression factor and that tells you about the passage of time. -Though you probably have to account for relativistic effects a bit, so far as passage of time goes. Sats, HELP! The movement of objects is a bit more complicated, essentially creating a single vector based on the clash of overlapping bubbles' frames of reference and compression factors. It's end up with objects going off at weird angles (Ex. two competing right angle vectors resulting in the object going off at 45 degrees), I think, but not necessarily being torn apart or the like. --- Realmatics of the all: Of course, none of this means anything unless you incorporate something to account for Cognitive aspects determining whether something is in or out of a bubble. Hence why I was careful to phrase everything in terms of "objects" up above, not space. This still leaves room for significant discussion of how exactly the surfaces of time bubbles work: Do they distend, or suck in people who touch them, or something else? Is there even really a "surface" worth talking about? It's an open question, so far as I can think of it at the moment. -- But, so far as the bubbles' impacts on all three Realms: Obviously they rely on Cognitive aspects to some extent to determine their occupancy. That judgement can likely to outsourced to the objects that enter the bubble: the bubble queries them to ask whether objects think they're "in" some given area, and gets a reply and reacts accordingly. So that deals with occupancy. But how do we model the actual effect these bubbles have on objects that are "in" them? I'll call back to the Spiritual again, though using a slightly different implementation than I've historically said (that model was rather off-the-cuff, to be fair). So we have two fundamental things to alter once an object enters the bubble: Its experience of the passage of time and its movement through space. Now, obviously, the two are rather closely linked the majority of the time. But we can't rely on that anymore because of frame-of-reference shenanigans. Time is easy enough: Just alter the object's passage through time to be t*X (the totality of all bubble effects) rather than the normal t. At the same time, though, we'll have to decouple the object's normal movement from the passage of time. Illuminating Thought Experiments: What this all means for movement: We can do vector math just as easily for multiple bubbles and three dimensions as we just did for one bubble and two dimensions, so let's leave that for now. How do we implement all of this, though? We may need some physics-brain for this. However, I do have a thought, if you'll bear with me for just a few more moments. Every object sees itself as at rest relative to itself, but that's boring and unuseful. More importantly for us, I think it fair to say that every object (at least on a planet, and likely elsewhere) also sees itself as in motion relative to something else. What if, then, it's that simple? Just reach into each object's Spiritual aspect and tweak it to change exactly one thing: From "you're moving at vector V relative to frame F" to "you're moving at vector V' relative to frame B", with V' as the object's vector as the bubble's frame of reference B perceives it. Can it be that simple? If "motion" is stored in (or at least tweak-able by) Spiritual aspects, I think it can. Maybe throw on a bit of extra oomph to make sure that any Physical or "real Spiritual" whining about "real" vectors is ignored until you get out of the bubble. Then just let the previous motion-rules come back into force, plus or minus any additional energy that was applied along any vector while the object was contained in the bubble. Satsuoni? Want to murder it? Conclusion: So that's about it. I'm very very open to suggestions on how to Realmatically solve the problem of motion once we decouple it from normal time; I do not have full confidence in my model as it stands. I'm also somewhat concerned that we may be using too much energy still, looking at individual objects rather than trying to change "Realm-space" to get general effects. P.S. A (rather horrifying, given how sure I've been about the way I model it, so far) thought: What if frame of reference isn't decided by the bubble, but by the object? So the object decides whether and how it's moving, then does a simple velocity*time accordingly. I don't think that'd end up very pretty (objects with different frames of reference could suddenly start colliding with each other or the like), but it could still work, technically. Any other thoughts?
  12. Hey guys/gals. I know you're probably getting tired of my time bubble threads by now, but now I have actual new information, so stuff might start happening. I asked some time bubble questions on the Q&A, and here are the answers: In general, a large object going through a time bubble is not going to notice. An object is either in or out, and it depends in part on how the object views itself. People inside the train would be inside of its influence, and wouldn't notice the bubble. The spear would go from one to the other, but would never be in both. We have definite confirmation of objects always being entirely in/out. We also have the example of a passenger in a train being unaffected by a Cadmium bubble which the train passes through. This might fall under my distension theory, with the train distending the bubble away from it's passengers, but we also have the intriguing statement that it "depends in part on how the object views itself." So a passenger who doesn't view him/herself as "on" the train might be affected? Or is that too broad an interpretation for "in part?" Don't worry, I won't subject you to a mega-post (yet). Theorize away!
  13. So, as some of you may know, I posted a "Definitive" (cough cough) Allomantic FTL Theory awhile back. I'm still working on how to achieve FTL, but a smaller thought (which may lead to FTL in the end) has come to mind just now. In-between being ruthlessly gutted by that foul entity known as SCIENCE, I tried to analyze exactly what happened when coherent objects were moving while partially inside and partially outside of a time bubble. In that thread, I theorized that the object retained its shape through the entire process, with the parts transiting the edge the bubble accelerating/decelerating (depending on the bubble) as they switch "time zones" and so pulling/pushing the rest of the object and therefore increasing or decreasing the speed of the object as a whole. Satsuoni kindly pointed out that that would result in nigh-instantaneous infinite acceleration (that's calc for you), the effects of which, I surmise, would mirror this description, and which we most definitely don't see in AoL. That still leaves us with the problem of how exactly the borders of time bubbles work, and what happens to coherent object that pass through them. In the interim, I have come up with two different solutions, which I will hopefully here explain satisfactorily and maybe even bring back to FTL or other practical applications. The Distension Theory: It's been theorized that the entire object transits simultaneously when it contacts the edge of the bubble, which I initially found somewhat absurd, but a thought has occurred to me as to how it could work in practice. Suppose that the edge of the bubble is not well defined, and that objects entering or exiting the bubble actually "distend" its surface (like a real bubble, as it were). When the bubble "pops", then the object which, up to this point, has been distending the bubble's surface is suddenly on the other side of the border with the bubble resuming its normal, roughly spherical dimensions. So if you throw a spear out of a bubble, it "bulges out" until enough of the spear (measured as a proportion of mass? Of length? As just a set amount of distension that any given bubble can support?) has gone far enough away from the center of the bubble, at which point the bubble's surface retracts until it no longer encompasses any of the the spear, returning to its normal dimensions after the spear has completely gone it's own way back into real time. This raises the interesting question of whether the "distension" of the bubble would reach objects near the spear on its way out, as opposed to only affecting the spear itself. If a grain of sand was 1mm away from the spear shaft's path in real time, would it be temporarily engulfed by the bubble? If the bubble retracts from the spear before it fully exits the bubble's normal volume, will objects around the spear but inside the bubble be affected? In this model, the ground, trains, building, etc. all actually compress the bubble a bit: for instance, a time bubble cast such as to touch the ground results in a flat-bottomed sphere, Perhaps even poking at the bubble with a stick would distend it inward, at least until the "grab that stick and show it who's boss" threshold is reached. This would actually be eminently testable: if speed bubbles couldn't reach through very solid objects--like, say, the wall of a building or a steel girder--to affect objects on the other side, then the theory looks basically correct. If they can, then we're back to simply having objects teleport into and out of the bubble's area of effect with no rhyme or reason. In the case of the girder, we might even be able to observe objects in the girder's "shadow" be outside the bubble while objects to its sides are inside of the bubble. I prefer distending the surface of the bubble to simply having objects be "in" the bubble and then "out" of the bubble all at once because, in the second case, we either have objects physically teleporting out of the bubble or half of an object being inexplicably in the wrong "time zone" despite being over a known border between the two. Distension also plays into the idea of a "bubble," though Brandon's use of that word may be entirely coincidental. As a ridiculously far-off theory that I'm just laying claim to for the heck of it without really putting much thought or reasoning behind it, what if the "pressure" of an atmosphere actually severely limits the size of time bubbles? So, once we're in space, bubbles suddenly grow to be kilometers's across, actually making "teleporting FTL" (see my "Definitive" thread) feasible. The Compression Theory: The other theory is a bit more out there and might be ruthlessly cut down by my eternal foe, Actual Understanding of the Physical World and Its Limitations. So we know that objects do not want to both be partially in and partially out of a time bubble; in two different time zones but still the same object which just happens to exist on both sides of the bubble, retaining the same shape with its pieces bound together by molecular bonds and whatnot. If that happens, we get infinite acceleration and the world blows up. However, what if the physical characteristics of an object actually change the border of a time bubble? So a 2m pole "compresses" into a shorter pole as it exits a speed bubble, the border compressing the pole as it leaves and then expanding the pole back, more slowly, on the "real time" side of the bubble in order to allow both sections to be traveling at the same speed, relative to their time zones. This allows the sections to be connected and still not pulling at each other. As the pole continues its transit, it compresses and expands so as to maintain this balance, and eventually leaves at its original 2m length. This could be tested by either observing and remembering (if it's that obvious) or photographing an object as it traverses the edge of a time bubble, assuming that the visual distortion isn't too bad. I might be able to hand-wave this with relativity stuff, "the ladder fits in the garage while you move it in, but not when you stop moving it," but I honestly don't know enough to make a good case. I feel as if I may have stumbled upon something here, with the borders of time bubbles pouring out some ridiculous energy to compress and expand objects as they pass, perhaps harness-able as "where the lost energy from thermodynamic issues goes in certain Allomantic interactions." I'll leave it to greater minds to decide. Conclusion: While I came up with the second theory first, I'm actually kind of liking the first one better. It may be that my baby-mind simply can't grasp the complexities and/or impossibilities present in the second theory, but I actually like how intuitive distension is for time bubbles. Even if neither of these theories is satisfactory, we still need to come up with some explanation for what exactly happens when objects enter or leave time bubbles. Partial entry/exit won't work, instantaneous complete entry/exit just doesn't make sense, and the solution has to be out there. TL;DR: You didn't think you'd actually get off that easily, did you?
×
×
  • Create New...